if k(x) = 3x, then f'(x)=? A. x³Ln3 B. 3xLn3 C. 3x/Lnx D. 3/3xLn3
The correct option is B .solution of given problem with the help of integrating the given function is 3xLn3
what is integration and function ?The area under a curve in a given range can be calculated mathematically via integration. To locate the region between the curve and the x-axis, it is necessary to find a function's antiderivative and evaluate it twice.
A function is a rule that gives each input value a distinct output value. It can be compared to a machine that processes inputs into outputs in accordance with a predetermined rule or formula.
According to given informationTo find f'(x), we need to take the derivative of f(x), where f(x) is the antiderivative of k(x).
Since k(x) = 3x, we can find f(x) by integrating 3x with respect to x:
f(x) = ∫ 3x dx = 3/2 x² + C
where C is a constant of integration.
Now we can find f'(x) by taking the derivative of f(x):
f'(x) = d/dx (3/2 x² + C) = 3x
Therefore, the answer is (B) 3xLn3. Option (A) is incorrect because there is no natural logarithm term in the derivative of f(x). Option (C) is incorrect because the derivative of 3x is 3, not 3/Ln(x). Option (D) is incorrect because there is no x in the denominator of the natural logarithm term.
To know more about integration Visit:
brainly.com/question/31433890
#SPJ1
Geometry.
Find the value of the variable
The value of the variable in the circle is x = 4.5.
How to find the value of the variable in the circle?If two secant segments shares an endpoint outside of the circle. The product of one secant segment and its external segment is equal to the product of the other secant segment and its external segment.
In this case, Use the theorem above, we can say:
5 * 18 = x * 20
90 = 20x
x = 90/20
x = 4.5
Thus, the value of the variable is x = 4.5.
Learn more about Secant-Secant Theorem on:
https://brainly.com/question/26340897
#SPJ1
A chemical balance needs to weigh 237g using weights of 50g, 20g, 10g and 1g. How many of each weight is needed?
Answer:
See below
Step-by-step explanation:
237/50 = 4 R 37
37/20 = 1 R 17
17 / 10 = 1 R 7
7 / 1 = 7 R 0
4 50gm's 1 20gm 1 10gm and 7 1g wts
The required number of weights used in chemical balance to weigh 237g is 4,1,1,7 respectively.
What is mathematical expression?
A mathematical expression expression in mathematical is defined as the collection of the numbers variables and functions by using signs like addition, subtraction, multiplication, and division.
Here, it is given that :
A chemical balance needs to weigh 237g using weights of 50g, 20g, 10g and 1g.
Now, let us find the number of weights for each weight type :
For 50g weight the maximum number of weights needed will be 4 that is :
50g x 4 = 200g
Now, for 20g weight the maximum number of weights needed will be 1 that is :
20g x 1 = 20g
Now, for 10g weight the maximum number of weights needed will be 1 that is :
10g x 1 = 10g
Now, for 1g weight the maximum number of weights needed will be 7 that is :
1g x 7 = 7g
Adding all the weights we get :
200g + 20g + 10g + 7g = 237g;
which is the required weight and the number of weights used is 4,1,1,7 respectively.
Read more about weights at:
https://brainly.com/question/16177712
#SPJ5
How many words with or without meaning each of 2 vowels and 3 consonants can be formed from the letters of the word "Honesty" ?
Answer:
The number of words with two vowels and 3 consonants from the letters of the word ‘HONESTY’ is 1200.
Step by step explanation:
A bean plant measures 1.5 cm and grows 0.1 cm per day. A tomato plant of 0.85 cm grows steadily too. How much can it grow each day knowing that it will have the same height as the bean plant in no more than 10 days?
Answer:
Let's call the growth rate of the tomato plant "g". We know that the height difference between the tomato plant and the bean plant decreases by 0.1 cm per day, so we can write the following equation:
1.5 - 0.85 - (10 * g) = 0
Solving for g:
g = (1.5 - 0.85) / 10 = 0.065 cm/day
So the tomato plant grows at a rate of 0.065 cm per day, and will reach the height of the bean plant in no more than 10 days.
The tomato plant will grow 0.165 cm per day.
Solution:
Given,
Height of bean plant = 1.5 cm
Height of tomato plant = 0.85 cm
The growth rate of bean plant = 0.1 cm/ day
The bean plant will grow 1.5 + (0.1* 10) in 10 days = 2.5 cm
So, the tomato plant will have to grow (2.5 - 0.85) cm in 10 days = 1.65 cm
Since, the tomato plant will grow steadily,
Hence, 1.65/ 10 = 0.165
Therefore, the tomato plant will grow at a rate of 0.165 cm per day
Learn more about how to simplify any expression here:
https://brainly.com/question/28036586
what about this one can someone help me NOT BOTSSSSSSS
Answer:
to be honest gang i dont know
Step-by-step explanation:
im just being honest
if you take away 25 from a number you will be left with two and halftimes 30. what is the number?
please help.. fast
Answer choices are:
A) interior
B) Alternative exterior
C) Corresponding
D) Complementary
Answer:
Complementary
Step-by-step explanation:
These 2 angles add to 90 degrees, making them complementary.
Answer:
complementary angle
Step-by-step explanation:
please mark me as brainlest
A sales person starts working 40 hours per week at a job with 2 options for being paid . Option A is an hourly wage of $19. Option B is a commission rate of 8% on weekly sales.
How much does the sales person need to sell in a given week to earn the same amount with each option?
A. $9,500
B. $4,750
C. $760
D. $320
Given, Option A: Hourly wage is $19 and the salesperson works 40 hours per week. So, he will earn in a week \(\sf = 19 \times 40 = \$760\)
Now, according to option b, he will get 8% commission on weekly sales.
Let. x = the amount of weekly sales.
To earn the same amount of option A, he will have to equal the 8% of x to $760
So, \(\sf \dfrac{8x}{100}=760\)
Or, \(\sf 8x= 76000\)
Or, \(\sf x= \dfrac{76000}{8}=9500\)
the salesman needs to make a weekly sales of $9,500 to earn the same amount with two options.
Can somebody help ill give you brainliest!!! and 20 points
Answer:
Genotype: 100% rr
phenotype: 100% white
Step-by-step explanation:
since there are no dominant genes the cross will definitely be a rr genotype
Is the event independent or overlapping:
A spinner has an equal chance of landing on each of its eight numbered regions. After spinning, what is the probability you land on region three and region six?
Mutually exclusive or independent:
A bag contains six yellow jerseys numbered 1-6. The bag also contains four purple jerseys numbered 1-4. You randomly pick a jersey. What is the probability it is purple or has a number greater than 5.
Mutually exclusive or overlapping:
A box of chocolates contains six milk chocolates and four dark chocolates. Two of the milk chocolates and three of the dark chocolates have peanuts inside. You randomly select and eat a chocolate. What is the probability that is is a milk chocolate or has no peanuts inside?
Mutually exclusive or independent:
You flip a coin and then roll a fair six sided die. What is the probability the coin lands on heads up and the die shows an even number?
The first question:
"A spinner has an equal chance of landing on each of its eight numbered regions. After spinning, what is the probability you land on region three and region six?"
Since the spinner has an equal chance of landing on each of its eight regions, the probability of landing on region three is 1/8, and the probability of landing on region six is also 1/8.
To find the probability of both events occurring (landing on region three and region six), you multiply the probabilities together:
P(landing on region three and region six) = P(landing on region three) * P(landing on region six) = (1/8) * (1/8) = 1/64.
Therefore, the probability of landing on both region three and region six is 1/64.
The events are mutually exclusive because it is not possible for the spinner to land on both region three and region six simultaneously.
--------------------------------------------------------------------------------------------------------------------------
The second question:
"A bag contains six yellow jerseys numbered 1-6. The bag also contains four purple jerseys numbered 1-4. You randomly pick a jersey. What is the probability it is purple or has a number greater than 5?"
To find the probability of either event occurring (purple or number greater than 5), we need to calculate the probabilities separately and then add them.
The probability of picking a purple jersey is 4/10 since there are four purple jerseys out of a total of ten jerseys.
The probability of picking a jersey with a number greater than 5 is 2/10 since there are two jerseys numbered 6 and above out of a total of ten jerseys.
To find the probability of either event occurring, we add the probabilities together:
P(purple or number greater than 5) = P(purple) + P(number greater than 5) = (4/10) + (2/10) = 6/10 = 3/5.
Therefore, the probability of picking a purple jersey or a jersey with a number greater than 5 is 3/5.
The events are overlapping since it is possible for the jersey to be both purple and have a number greater than 5.
--------------------------------------------------------------------------------------------------------------------------
The third question:
"A box of chocolates contains six milk chocolates and four dark chocolates. Two of the milk chocolates and three of the dark chocolates have peanuts inside. You randomly select and eat a chocolate. What is the probability that it is a milk chocolate or has no peanuts inside?"
To find the probability of either event occurring (milk chocolate or no peanuts inside), we need to calculate the probabilities separately and then add them.
The probability of selecting a milk chocolate is 6/10 since there are six milk chocolates out of a total of ten chocolates.
The probability of selecting a chocolate with no peanuts inside is 7/10 since there are seven chocolates without peanuts out of a total of ten chocolates.
To find the probability of either event occurring, we add the probabilities together:
P(milk chocolate or no peanuts inside) = P(milk chocolate) + P(no peanuts inside) = (6/10) + (7/10) = 13/10.
Therefore, the probability of selecting a milk chocolate or a chocolate with no peanuts inside is 13/10.
The events are mutually exclusive since a chocolate cannot be both a milk chocolate and have no peanuts inside simultaneously.
--------------------------------------------------------------------------------------------------------------------------
The fourth question:
"You flip a coin and then roll a fair six-sided die. What is the probability the coin lands heads up and the die shows an even number?"
The probability of the coin landing heads up is 1/2 since there are two possible outcomes (heads or tails) and they are equally likely.
The probability of rolling an even number on the die is 3
/6 or 1/2 since there are three even numbers (2, 4, and 6) out of a total of six possible outcomes.
To find the probability of both events occurring (coin lands heads up and die shows an even number), we multiply the probabilities together:
P(coin lands heads up and die shows an even number) = P(coin lands heads up) * P(die shows an even number) = (1/2) * (1/2) = 1/4.
Therefore, the probability of the coin landing heads up and the die showing an even number is 1/4.
The events are independent since the outcome of flipping the coin does not affect the outcome of rolling the die.
\(\huge{\mathfrak{\colorbox{black}{\textcolor{lime}{I\:hope\:this\:helps\:!\:\:}}}}\)
♥️ \(\large{\underline{\textcolor{red}{\mathcal{SUMIT\:\:ROY\:\:(:\:\:}}}}\)
which rectangle has the greatest area?
A: Rectangle A that is 3.25 inches wide and 6.1 inches long or
B: rectangle B that is 4.06 inches wide and 5.009 inches long.
Answer:
Rectangle B
Step-by-step explanation:
Rectangle A:
L × B
3.25 × 6.1
19.825 inches
Rectangle B:
L × B
4.06 × 5.009
20.33654 inches
Determine the domain and the range of the given graph of a function.
The domain of the graph of the function is?
(Type your answer in interval notation.)
The domain and the range of the graph are
Domain = [-6, -1] U [1, 5}Range = [-3, 7]Calculating the domain and range of the graphFrom the question, we have the following parameters that can be used in our computation:
The graph
The rule of a function is that
The domain is the x valuesThe range is the f(x) valuesUsing the above as a guide, we have the following:
Domain = [-6, -1] U [1, 5}
And, we have
Range = [-3, 7]
Read more about domain and range at
brainly.com/question/27910766
#SPJ1
Reasoning 1f 60% of the registered voters cast 202,800 votes in an election, how many registered voters
of registered voters compare in the two elections? Show your work.
There are registered voters. (Type a whole number.)
Answer:
338,000
Step-by-step explanation:
202800 is to 338000 like 60% is to 100%
hope that helps lol (did it with my mouse so looks awful)
Sales tax is calculated as a percentage of the sales price. If sales tax is 6%, what is the sales tax on clothing that costs $180? (3 points)
Answer:
10.80
Step-by-step explanation:
Sales price x sales tax rate = sales tax
180 x .06 (6%) = 10.80
The amount of sales tax for the clothing that costs $180 is $10.8.
Given that,
Cost of a clothing = $180
We have to find the sales tax amount.
Sales tax is calculated as a percentage of the sales price.
Here,
Sales tax percentage = 6%
In order to find the actual sales tax amount, we have to multiply the cost with the percentage of sales tax.
Sales tax amount = 6% of 180
= (6/100) 180
= 0.6 × 18
= 10.8
Hence the sales tax amount is $10.8.
Learn more about Sales Tax here :
https://brainly.com/question/29442509
#SPJ6
how do i solve this abs what is the ansswer
Answer:
3 -4 5
Step-by-step explanation:
it just works trust
Pls help
Which of these rational number's is the product of the other 6:
2/99, 2/3, 5/7, 2 1/3, 2 1/4, 10/11, 18.
Answer:
10/11
Step-by-step explanation:
Brianna makes a model of the Eiffel Tower for a school project. Her model is 25 centimeters wide at the bottom
If Brianna used a scale of 1 centimeter for every 5 meters, how vide is the actual Efe Towet?
The actual Eiffel Tower is 125 meters wide .
What is Multiplication?
Along with addition, subtraction, and division, multiplication is one of the four basic arithmetic operations. In mathematics, multiplication is the repeated addition of groups of equal size.
Here given, Brianna makes a model of the Eiffel Tower for a school project.
It is also given that the model is 25 cm wide at the bottom and Brianna used a scale of 1 centimeter for every 5 meters.
By multiplication, we can find the actual length of Eiffel tower
Here 1 centimeter for every 5 meters.
So, 25 centimeter means (5×25) = 125 meters.
Therefore, the actual length of the Eiffel tower is 125 meters.
Learn more about multiplication here,
https://brainly.com/question/19267385
#SPJ1
"Correct question is "Brianna makes a model of the Eiffel Tower for a school project. Her model is 25 centimeters wide at the bottom
If Brianna used a scale of 1 centimeter for every 5 meters, how wide is the actual Eiffel Tower?"
A water cooler has a storage capacity of 50 litres . If each student on an average consumes 750ml of water. Then how many times cooler tank has to be filled to quench the thirst of 1200 student of school
The water cooler tank has to be filled 18 times to quench the thirst of 1200 students.
We have
Number of students= 1200
Water consumption per student= 750 ml
So, Total water consumption
= Number of students × Water consumption per student
= 1200 × 750 ml
= 900000 ml
= 900 Liter
Now, the time cooler tank has to be filled to quench the thirst
= 900 / 50
= 18 times
Thus, the water cooler tank has to be filled 18 times to quench the thirst of 1200 students.
Learn more about Unitary Method here:
https://brainly.com/question/22056199
#SPJ1
A bike path is 12.5 miles. Dominic is 70% of the way to the end. How far is Dominic on the path?
The percentage of the distance Dominic is on the path is his distance
travelled as a fraction of the total distance when equivalent to 100.
The distance he has travelled on the path is 8.75 miles.
The given parameter are;
Length of Dominic's path = 12.5 miles
The length of the way Dominic is to the end = 70%
Required:
The distance Dominic has travelled on the path
The distance Dominic has travelled, D is 70% of 12.5 miles.
70% of 12.5 miles = 70% × 12.5 miles = 8.75 miles
The distance Dominic has travelled on the path, D = 8.75 miles
Learn more about Distance here:
brainly.com/question/24344086
#SPJ9
probably very easy but i do need some help asap
my calculations are not giving me the answer i'm supposed to receive which is
9y^3(3-y)
Answer:
\(\sf 9y^3(3-y)\)
Step by step explanation:
\(\sf 27y^3-9y^4\)
Factor out common term 9 :
\(\sf \implies 9(3y^3-y^4)\)
Apply exponent rule \(\sf a^{bc}=a^{b+c}=a^ba^c\) :
\(\sf \implies 9(3y^3-y^{1+3})\)
\(\sf \implies 9(3y^3-y^1y^3)\)
Factor out common term \(\sf y^3\):
\(\sf \implies 9y^3(3-y^1)\)
Apply exponent rule \(\sf a^1=a\) :
\(\sf \implies 9y^3(3-y)\)
3.Rhombi have 4 sides that are congruent.
True
O
False
Answer:
true
Step-by-step explanation:
. Order the following numbers from least to greatest: 3√2 , √3 − 1, √19 + 1, 6,
2√10 ÷ 5 and √14.
pls someone help me
The required order from least to greatest is √3 − 1, 2√10 ÷ 5, √14, 3√2, √19 + 1 and 6
What is ascending order?An arrangement of numbers in which the numbers are arranged from smallest to greatest numbers is called ascending order.
Given that, some numbers, 3√2, √3 − 1, √19 + 1, 6, 2√10 ÷ 5 and √14.
We need to order them from least to greatest,
3√2 = 4.24
√3 − 1 = 0.73
√19 + 1 = 5.35
6
2√10 ÷ 5 = 1.26
√14 = 3.74
The order from least to greatest is :-
0.73, 1.26, 3.74, 4.24, 5.35 and 6
i.e.
√3 − 1, 2√10 ÷ 5, √14, 3√2, √19 + 1 and 6
Hence, the required order from least to greatest is √3 − 1, 2√10 ÷ 5, √14, 3√2, √19 + 1 and 6
Learn more about ascending order, click;
https://brainly.com/question/28762233
#SPJ9
\(77 \times 5 \: equal \: what \\ \)
what is the answer
Answer:
385 is the result of multiply 77 by 5
PLEASE ANSWER ASAP!!!!!
Answer:
\(\huge\boxed{\sf r = 5}\)
Step-by-step explanation:
Given that,
7(q + 5) = (q + r)7Distribute7q + 35 = 7q + 7r
Subtract 7q from both sides7q - 7q + 35 = 7q - 7q + 7r
35 = 7r
Divide both sides by 735/7 = r
5 = r
OR
r = 5\(\rule[225]{225}{2}\)
Answer:
r = 5
Step-by-step explanation:
Given statement,
→ 7(q + 5) is equivalent to (q + r)7.
Forming the equation,
→ 7(q + 5) = 7(q + r)
Now we have to,
→ Find the required value of r.
Then the value of r will be,
→ 7(q + 5) = 7(q + r)
Applying Distributive property:
→ 7(q) + 7(5) = 7(q) + 7(r)
→ 7q + 35 = 7q + 7r
Cancelling 7q from both sides:
→ 35 = 7r
→ 7r = 35
Dividing the RHS with number 7:
→ r = 35/7
→ [ r = 5 ]
Therefore, the value of r is 5.
I need help right now!!!
1. Figure STUV is congruent to Figure KLMN because rigid motions can be used to map Figure STUV onto Figure KLMN.
2. Figure STUV is also similar to Figure KLMN because rigid motions and/or dilations can be used to map Figure STUV onto Figure KLMN. The scale factor is 2.
What does it mean for figures to be congruent?When it is said that two figures are congruent, like the ones shown on the graph, This means that they have the same shape and size.
Similar figures have the same shape, but they may have different sizes and be located in different positions.
Find more exercises on congruent figures;
https://brainly.com/question/29193622
#SPJ1
Convert 50°F to degrees Celsius.
If necessary, round your answer to the nearest tenth of a degree.
Here are the formulas.
C=5/9(F-32)
F=9/5C+32
Answer:
10°C
Step-by-step explanation:
F = 50
C = 5/9 * (F - 32) = 5/9 * (50 - 32) = 5/9 * 18 = 5 * 2 = 10
What is the slope of the line on the graph?
Enter your answer in the box.
Answer:
6
Step-by-step explanation:
The find the slope of a line given any two points, we can use the slope formula given by:
\(\displaystyle m=\frac{y_2-y_1}{x_2-x_1}\)
We can choose the two points (0, -1) and (1, 5) from the graph.
Let (0, -1) be (x₁, y₁) and let (1, 5) be (x₂, y₂). So:
\(\displaystyle m=\frac{(5)-(-1)}{(1)-0}=\frac{6}{1}=6\)
Hence, the slope of the line is 6.
slope is the ratio of vertical increase to horizontal increase
Learning
Diagnostic
Analytics
See your levels
Professor Gomez weighed two pieces of metal for an experiment. The piece of iron weighed
5/12 of a pound and the piece of aluminum weighed 1/6 of a pound. How much more did the
piece of iron weigh than the piece of aluminum?
Write your answer as a fraction or as a whole or mixed number.
pounds
Submit
I don't know this
In the question, we are given the following parameters.
Weight of iron = 5/12 of a pound
Weight of aluminium = 1/6 of a pound.
Explanation
We can find how much more did the piece of iron weigh than the piece of aluminum by subtracting the fractions that represents their weight. Thereofore we have
\(undefined\)FREE BRAINLIST NEED HELP ASAP
Answer:
The answer is C
Step-by-step explanation:
Trust me I made a graph to organize the numbers